(20 intermediate revisions by 2 users not shown)
Line 1: Line 1:
= [[ECE PhD Qualifying Exams|ECE Ph.D. Qualifying Exam]] in "Automatic Control" (AC) =
+
[[Category:ECE]]
 +
[[Category:QE]]
 +
[[Category:CNSIP]]
 +
[[Category:problem solving]]
 +
[[Category:automatic control]]
 +
[[Category:optimization]]
  
= Question 3, Part 3, August 2011  =
+
= [[ECE PhD Qualifying Exams|ECE Ph.D. Qualifying Exam]] in "Automatic Control" (AC)  =
 +
 
 +
=  [[ECE-QE_AC3-2011|Question 3, August 2011]],  Part 3 =
  
 
:[[ECE-QE AC3-2011 solusion-1|Part 1]],[[ECE-QE AC3-2011 solusion-2|2]],[[ECE-QE_AC3-2011_solusion-3|3]],[[ECE-QE AC3-2011 solusion-4|4]],[[ECE-QE AC3-2011 solusion-5|5]]
 
:[[ECE-QE AC3-2011 solusion-1|Part 1]],[[ECE-QE AC3-2011 solusion-2|2]],[[ECE-QE_AC3-2011_solusion-3|3]],[[ECE-QE AC3-2011 solusion-4|4]],[[ECE-QE AC3-2011 solusion-5|5]]
Line 19: Line 26:
 
----
 
----
  
The '''Fundamental Theorem of Linear Programming''' that one of the basic feasible solutions is an optimal solution. So we generate all possible basic feasible solutions and select from them the optimal one.
+
'''Theorem:'''
 +
 
 +
The '''Fundamental Theorem of Linear Programming''' that one of the basic feasible solutions is an optimal solution. 
  
 
----
 
----
Line 32: Line 41:
 
<math>\Rightarrow x_{2}-x_{3}=4</math>  
 
<math>\Rightarrow x_{2}-x_{3}=4</math>  
  
<math>\text{It is equivalent to  min }  x_{1}+3x_{2}-4x_{3}
+
<span class="texhtml">It is equivalent to  min ''x''<sub>1</sub> + 3''x''<sub>2</sub> − 4''x''<sub>3</sub> = 5 2''x''<sub>2</sub> + ''x''<sub>3</sub> + 3''x''<sub>2</sub> − 4''x''<sub>3</sub> = ''x''<sub>2</sub> − 3''x''<sub>3</sub> + 5,</span>  
=5-2x_{2}+x_{3}+3x_{2}-4x_{3} = x_{2}-3x_{3}+5, x_{2}\geq0, x_{3}\leq0</math>  
+
  
<math>x_{2}-3x_{3}+5 = x_{2}-x_{3}-2x_{3}+5=9-2x_{3}\geq9</math>  
+
&nbsp; &nbsp; &nbsp; &nbsp; &nbsp; &nbsp; &nbsp; &nbsp; &nbsp; &nbsp; &nbsp; &nbsp; &nbsp; &nbsp; &nbsp; &nbsp;&nbsp;<math>x_{2}\geq0, x_{3}\leq0</math>
 +
 
 +
<math>x_{2}-3x_{3}+5 = x_{2}-x_{3}-2x_{3}+5=9-2x_{3}\geq9</math>&nbsp; &nbsp;<math>\color{green}  \text{constrain:  } x_{3}\leq0 \Rightarrow x_{3}=0</math>  
  
 
<math>\text{Equivalently, } -x_{1}-3x_{2}+4x_{3}\leq-9</math>  
 
<math>\text{Equivalently, } -x_{1}-3x_{2}+4x_{3}\leq-9</math>  
  
<math>\text{Equality is satisfied when } x_{3}=0, x_{2} =4, x_{1}=5-2\times4=-3</math>  
+
<math>\text{Equality is satisfied when } x_{3}=0, x_{2} =4+0=4, x_{1}=5-2\times4=-3</math>  
  
 
<math>\Rightarrow \left\{\begin{matrix}
 
<math>\Rightarrow \left\{\begin{matrix}
Line 45: Line 55:
 
x_{2}=4\\  
 
x_{2}=4\\  
 
x_{3}=0
 
x_{3}=0
\end{matrix}\right.</math>  
+
\end{matrix}\right.</math>&nbsp; &nbsp; &nbsp; &nbsp; &nbsp;&nbsp;<math>\color{green} \text{The answer is correct.}</math>  
  
<math>\color{green} \text{This solution is not using the Fundamental Theorem of Linear Programming}</math>
+
<math>\color{green} \text{But, this solution is NOT using the Fundamental Theorem of LP.}</math>  
  
 
----
 
----
Line 53: Line 63:
 
<math>\color{blue}\text{Solution 2:}</math>  
 
<math>\color{blue}\text{Solution 2:}</math>  
  
One of the basic feasible solution is an optimal solution.<br><font face="serif"></font><math>\text{The equality constraints can be represented in the form } Ax=b, A= \begin{bmatrix}
+
One of the basic feasible solution is an optimal solution.<br><font face="serif"></font><span class="texhtml">The equality constraints can be represented in the form ''A'''x'''''<b> = ''b'',</b></span>
 +
 
 +
<math>A=\begin{bmatrix}
 +
1 & 2 & -1\\
 +
2 & 3 & -1
 +
\end{bmatrix}=\begin{bmatrix}
 
a_{1}&  
 
a_{1}&  
 
a_{2}&  
 
a_{2}&  
 
a_{3}
 
a_{3}
 +
\end{bmatrix}; b=\begin{bmatrix}
 +
5\\
 +
6
 
\end{bmatrix}</math>  
 
\end{bmatrix}</math>  
  
<math>\text{The fist basis candidate is } \begin{pmatrix}
+
<math>\text{The first basis candidate is } \begin{pmatrix}
 
a_{1} &  
 
a_{1} &  
 
a_{2}  
 
a_{2}  
 
\end{pmatrix}</math>  
 
\end{pmatrix}</math>  
  
<math>\left [ A|b \right ] =\begin{bmatrix}  
+
&nbsp; &nbsp; &nbsp; &nbsp;&nbsp;<math>\left [ A|b \right ] =\begin{bmatrix}  
 
1 & 2 & -1 & 5 \\  
 
1 & 2 & -1 & 5 \\  
 
2 & 3 & -1 & 6  
 
2 & 3 & -1 & 6  
Line 72: Line 90:
 
\end{bmatrix}</math>  
 
\end{bmatrix}</math>  
  
<math>x^{\left( 1 \right)}= \begin{bmatrix}
+
&nbsp; &nbsp; &nbsp; &nbsp;&nbsp;<math>x^{\left( 1 \right)}= \begin{bmatrix}
 
-3 & 4 & 0
 
-3 & 4 & 0
\end{bmatrix} \text{ is BFS. } f_{1}=-9</math>  
+
\end{bmatrix}^{T} \text{ is a BFS. } f_{1}=-9</math>  
  
 
<math>\text{The second basis candidate is } \begin{pmatrix}
 
<math>\text{The second basis candidate is } \begin{pmatrix}
Line 81: Line 99:
 
\end{pmatrix}</math>  
 
\end{pmatrix}</math>  
  
<math>\left [ A|b \right ] =\begin{bmatrix}  
+
&nbsp; &nbsp; &nbsp; &nbsp;&nbsp;<math>\left [ A|b \right ] =\begin{bmatrix}  
 
1 & 2 & -1 & 5 \\  
 
1 & 2 & -1 & 5 \\  
 
2 & 3 & -1 & 6  
 
2 & 3 & -1 & 6  
Line 89: Line 107:
 
\end{bmatrix}</math>  
 
\end{bmatrix}</math>  
  
<math>x^{\left( 2 \right)}= \begin{bmatrix}
+
&nbsp; &nbsp; &nbsp; &nbsp;&nbsp;<math>x^{\left( 2 \right)}= \begin{bmatrix}
 
0 & 1 & -3
 
0 & 1 & -3
\end{bmatrix} \text{ is BFS. } f_{2}=-15</math>  
+
\end{bmatrix}^{T} \text{ is a BFS. } f_{2}=-15</math>  
  
 
<math>\text{The third basis candidate is } \begin{pmatrix}
 
<math>\text{The third basis candidate is } \begin{pmatrix}
Line 98: Line 116:
 
\end{pmatrix}</math>  
 
\end{pmatrix}</math>  
  
<math>\left [ A|b \right ] =\begin{bmatrix}  
+
&nbsp; &nbsp; &nbsp; &nbsp;&nbsp;<math>\left [ A|b \right ] =\begin{bmatrix}  
 
1 & 2 & -1 & 5 \\  
 
1 & 2 & -1 & 5 \\  
 
2 & 3 & -1 & 6  
 
2 & 3 & -1 & 6  
Line 106: Line 124:
 
\end{bmatrix}</math>  
 
\end{bmatrix}</math>  
  
<math>x^{\left( 2 \right)}= \begin{bmatrix}
+
&nbsp; &nbsp; &nbsp; &nbsp;&nbsp;<math>x^{\left( 2 \right)}= \begin{bmatrix}
 
1 & 0 & -5
 
1 & 0 & -5
\end{bmatrix} \text{ is BFS. } f_{3}=-21</math>  
+
\end{bmatrix}^{T} \text{ is a BFS. } f_{3}=-21</math>  
  
<math>\because f_{1}>f_{2}>f_{3}</math>  
+
<math>\because f_{1}>f_{2}>f_{3} \text{ , where } f_{1}=-9 \text{ is maximal.}</math>  
  
 
<math>\therefore \text{The optimal solution is } x^{*}=\begin{bmatrix}
 
<math>\therefore \text{The optimal solution is } x^{*}=\begin{bmatrix}
 
-3 & 4 & 0  
 
-3 & 4 & 0  
 
\end{bmatrix}  \text{ with objective value } -9</math>  
 
\end{bmatrix}  \text{ with objective value } -9</math>  
 +
 +
<math>\color{green} \text{This solution use the Fundamental Theorem of Linear Programming. }</math>
 +
 +
<math>\color{green} \text{All possible basic feasible solutions are generated }</math>
 +
 +
&nbsp; &nbsp; &nbsp; &nbsp; &nbsp; &nbsp; &nbsp;&nbsp;<math>\color{green} \text{ and from which the optimal one is selected.}</math>
 +
 +
----
 +
 +
<math>\color{blue} \text{Related Problem: Solve the following linear programming problem,}</math>
 +
 +
minimize &nbsp;&nbsp;<span class="texhtml">3''x''<sub>1</sub> + ''x''<sub>2</sub> + ''x''<sub>3</sub></span>
 +
 +
subject to &nbsp;<span class="texhtml">''x''<sub>1</sub> + ''x''<sub>3</sub> = 4</span>
 +
 +
&nbsp; &nbsp; &nbsp; &nbsp; &nbsp; &nbsp; &nbsp; &nbsp; &nbsp;&nbsp;<span class="texhtml">''x''<sub>2</sub> − ''x''<sub>3</sub> = 2</span>
 +
 +
&nbsp; &nbsp; &nbsp; &nbsp; &nbsp; &nbsp; &nbsp; &nbsp; &nbsp;&nbsp;<math>x_{1}\geq0,x_{2}\geq0,x_{3}\geq0,</math>
 +
 +
<math>\color{blue}\text{Solution :}</math>
 +
 +
<span class="texhtml">The equality constraints can be represented in the form ''A'''x'''''<b> = ''b'',</b></span>'''<br> '''
 +
 +
<math>A= \begin{bmatrix}
 +
1 & 0 & 1\\
 +
0 & 1 & -1
 +
\end{bmatrix}=\begin{bmatrix}
 +
a_{1}&
 +
a_{2}&
 +
a_{3}
 +
\end{bmatrix}; b=\begin{bmatrix}
 +
4\\
 +
2
 +
\end{bmatrix}</math>
 +
 +
<math>\text{The first basis candidate is } \begin{pmatrix}
 +
a_{1} &
 +
a_{2}
 +
\end{pmatrix}</math><br>
 +
 +
&nbsp; &nbsp; &nbsp; &nbsp; &nbsp;<math>\text{The corresponding basic solution is } x^{\left( 1 \right)}= \begin{bmatrix}
 +
4 & 2 & 0
 +
\end{bmatrix}^{T} \text{ is a BFS.}</math><br>
 +
 +
&nbsp; &nbsp; &nbsp; &nbsp; &nbsp;<span class="texhtml">The objective function value is ''f''<sub>1</sub> = 14</span>
 +
 +
<math>\text{The second basis candidate is } \begin{pmatrix}
 +
a_{1} &
 +
a_{3}
 +
\end{pmatrix}</math><br>
 +
 +
&nbsp; &nbsp; &nbsp; &nbsp;&nbsp;<math>\text{The corresponding basic solution is } x^{\left( 2 \right)}= \begin{bmatrix}
 +
6 & 0 & -2
 +
\end{bmatrix}^{T} \text{, which is NOT a BFS.}</math>&nbsp; &nbsp; &nbsp; &nbsp;
 +
 +
<math>\text{The third basis candidate is } \begin{pmatrix}
 +
a_{2} &
 +
a_{3}
 +
\end{pmatrix}</math>
 +
 +
&nbsp; &nbsp; &nbsp; &nbsp; &nbsp;<math>\text{The corresponding basic solution is } x^{\left( 3 \right)}= \begin{bmatrix}
 +
0 & 6 & 4
 +
\end{bmatrix}^{T} \text{, which is a BFS.}</math>
 +
 +
&nbsp; &nbsp; &nbsp; &nbsp; &nbsp;<span class="texhtml">The objective function value is ''f''<sub>3</sub> = 10</span>
 +
 +
<font color="#ff0000"><span style="font-size: 17px;">'''<math>\therefore \text{ the optimal solution is } x^{ * }= \begin{bmatrix} 0 & 6 & 4 \end{bmatrix}^{T}</math>'''</span></font>
 +
 +
<br>
  
 
----
 
----
Line 130: Line 217:
 
----
 
----
  
<br> [[ECE PhD Qualifying Exams|Back to ECE Qualifying Exams (QE) page]]
+
<br> [[ECE PhD Qualifying Exams|Back to ECE Qualifying Exams (QE) page]]
 
+
[[Category:ECE]] [[Category:QE]] [[Category:Automatic_Control]] [[Category:Problem_solving]]
+

Latest revision as of 10:10, 13 September 2013


ECE Ph.D. Qualifying Exam in "Automatic Control" (AC)

Question 3, August 2011, Part 3

Part 1,2,3,4,5

 $ \color{blue}\text{3. } \left( \text{20 pts} \right) \text{ Solve the following linear program, } $

maximize    − x1 − 3x2 + 4x3

subject to    x1 + 2x2x3 = 5

                   2x1 + 3x2x3 = 6

                   $ x_{1} \text{ free, } x_{2}\geq0, x_{3}\leq0. $


Theorem:

The Fundamental Theorem of Linear Programming that one of the basic feasible solutions is an optimal solution. 


$ \color{blue}\text{Solution 1:} $

$ \left.\begin{matrix} x_{1}+2x_{2}-x_{3}=5 \\ 2x_{1}+3x_{2}-x_{3}=6 \end{matrix}\right\}\Rightarrow x_{1}=5-2x_{2}+x_{3}=3-\frac{3}{2}x_{2}+\frac{1}{2}x_{3} $

$ \Rightarrow x_{2}-x_{3}=4 $

It is equivalent to min x1 + 3x2 − 4x3 = 5 − 2x2 + x3 + 3x2 − 4x3 = x2 − 3x3 + 5,

                                $ x_{2}\geq0, x_{3}\leq0 $

$ x_{2}-3x_{3}+5 = x_{2}-x_{3}-2x_{3}+5=9-2x_{3}\geq9 $   $ \color{green} \text{constrain: } x_{3}\leq0 \Rightarrow x_{3}=0 $

$ \text{Equivalently, } -x_{1}-3x_{2}+4x_{3}\leq-9 $

$ \text{Equality is satisfied when } x_{3}=0, x_{2} =4+0=4, x_{1}=5-2\times4=-3 $

$ \Rightarrow \left\{\begin{matrix} x_{1}=-3\\ x_{2}=4\\ x_{3}=0 \end{matrix}\right. $          $ \color{green} \text{The answer is correct.} $

$ \color{green} \text{But, this solution is NOT using the Fundamental Theorem of LP.} $


$ \color{blue}\text{Solution 2:} $

One of the basic feasible solution is an optimal solution.
The equality constraints can be represented in the form Ax = b,

$ A=\begin{bmatrix} 1 & 2 & -1\\ 2 & 3 & -1 \end{bmatrix}=\begin{bmatrix} a_{1}& a_{2}& a_{3} \end{bmatrix}; b=\begin{bmatrix} 5\\ 6 \end{bmatrix} $

$ \text{The first basis candidate is } \begin{pmatrix} a_{1} & a_{2} \end{pmatrix} $

        $ \left [ A|b \right ] =\begin{bmatrix} 1 & 2 & -1 & 5 \\ 2 & 3 & -1 & 6 \end{bmatrix} = \cdots = \begin{bmatrix} 1 & 0 & 1 & -3 \\ 0 & 1 & -1 & 4 \end{bmatrix} $

        $ x^{\left( 1 \right)}= \begin{bmatrix} -3 & 4 & 0 \end{bmatrix}^{T} \text{ is a BFS. } f_{1}=-9 $

$ \text{The second basis candidate is } \begin{pmatrix} a_{2} & a_{3} \end{pmatrix} $

        $ \left [ A|b \right ] =\begin{bmatrix} 1 & 2 & -1 & 5 \\ 2 & 3 & -1 & 6 \end{bmatrix} = \cdots = \begin{bmatrix} 1 & 0 & 1 & -3 \\ 1 & 1 & 0 & 1 \end{bmatrix} $

        $ x^{\left( 2 \right)}= \begin{bmatrix} 0 & 1 & -3 \end{bmatrix}^{T} \text{ is a BFS. } f_{2}=-15 $

$ \text{The third basis candidate is } \begin{pmatrix} a_{1} & a_{3} \end{pmatrix} $

        $ \left [ A|b \right ] =\begin{bmatrix} 1 & 2 & -1 & 5 \\ 2 & 3 & -1 & 6 \end{bmatrix} = \cdots = \begin{bmatrix} 0 & 1 & 1 & 4 \\ 1 & 1 & 0 & 1 \end{bmatrix} $

        $ x^{\left( 2 \right)}= \begin{bmatrix} 1 & 0 & -5 \end{bmatrix}^{T} \text{ is a BFS. } f_{3}=-21 $

$ \because f_{1}>f_{2}>f_{3} \text{ , where } f_{1}=-9 \text{ is maximal.} $

$ \therefore \text{The optimal solution is } x^{*}=\begin{bmatrix} -3 & 4 & 0 \end{bmatrix} \text{ with objective value } -9 $

$ \color{green} \text{This solution use the Fundamental Theorem of Linear Programming. } $

$ \color{green} \text{All possible basic feasible solutions are generated } $

              $ \color{green} \text{ and from which the optimal one is selected.} $


$ \color{blue} \text{Related Problem: Solve the following linear programming problem,} $

minimize   3x1 + x2 + x3

subject to  x1 + x3 = 4

                  x2x3 = 2

                  $ x_{1}\geq0,x_{2}\geq0,x_{3}\geq0, $

$ \color{blue}\text{Solution :} $

The equality constraints can be represented in the form Ax = b,

$ A= \begin{bmatrix} 1 & 0 & 1\\ 0 & 1 & -1 \end{bmatrix}=\begin{bmatrix} a_{1}& a_{2}& a_{3} \end{bmatrix}; b=\begin{bmatrix} 4\\ 2 \end{bmatrix} $

$ \text{The first basis candidate is } \begin{pmatrix} a_{1} & a_{2} \end{pmatrix} $

         $ \text{The corresponding basic solution is } x^{\left( 1 \right)}= \begin{bmatrix} 4 & 2 & 0 \end{bmatrix}^{T} \text{ is a BFS.} $

         The objective function value is f1 = 14

$ \text{The second basis candidate is } \begin{pmatrix} a_{1} & a_{3} \end{pmatrix} $

        $ \text{The corresponding basic solution is } x^{\left( 2 \right)}= \begin{bmatrix} 6 & 0 & -2 \end{bmatrix}^{T} \text{, which is NOT a BFS.} $       

$ \text{The third basis candidate is } \begin{pmatrix} a_{2} & a_{3} \end{pmatrix} $

         $ \text{The corresponding basic solution is } x^{\left( 3 \right)}= \begin{bmatrix} 0 & 6 & 4 \end{bmatrix}^{T} \text{, which is a BFS.} $

         The objective function value is f3 = 10

$ \therefore \text{ the optimal solution is } x^{ * }= \begin{bmatrix} 0 & 6 & 4 \end{bmatrix}^{T} $



Automatic Control (AC)- Question 3, August 2011

Go to



Back to ECE Qualifying Exams (QE) page

Alumni Liaison

Ph.D. 2007, working on developing cool imaging technologies for digital cameras, camera phones, and video surveillance cameras.

Buyue Zhang